LSAT and Law School Admissions Forum

Get expert LSAT preparation and law school admissions advice from PowerScore Test Preparation.

 srcline@noctrl.edu
  • Posts: 243
  • Joined: Oct 16, 2015
|
#23964
Hello

Can someone please explain why D is not the correct answer. So this is Justify the Conclusion question. So we are looking for an a.c. that is sufficient for the conclusion to be true. JC formula: Premise + Answer choice = conclusion

many mtn. climbers regard climbing mt. everest as ult. ach. +D (profound spiritual exp. can be achieved w/o undergoing the serious danger involved in mountain climbing = con.(climbers shouldn't climb mt. eve. b/c risk of death/ injury and climbers dispel reports of notion of gaining spiritual discovery.

So this a.c. doesn't really make sense.

SO this is the reason why B is correct?

Thankyou
Sarah
 Clay Cooper
PowerScore Staff
  • PowerScore Staff
  • Posts: 241
  • Joined: Jul 03, 2015
|
#24020
Hi Sarah,

Thanks for this question.

Answer choice D is bad because it misses the point. Remember to focus very specifically on the conclusion: Climbers shouldn't attempt to climb Everest (the second half of that sentence, where we learn that climbing Everest involves significant risk of injury or death, is just another premise).

So now, when we apply the Justify Formula, here is what we have:
Evidence + Answer Choice = Conclusion
Climbing Everest is likely to hurt or kill you + ( ) = Climber's shouldn't attempt it

Now we can prephrase: we need an answer choice that allows us, when we know something is dangerous, to prove that it should not be undertaken; answer choice B does that.

Answer choice D misses the point by focusing on the spiritual experience; notice that the conclusion doesn't mention spiritual experiences, and it is not necessary that we address them in order to prove the conclusion. The correct answer choice does mention them, it is true. But it doesn't have to - B would be just as correct if it simply said 'Dangerous activities should be avoided' and stopped there.

Focus on proving the conclusion; that is all you have to do in a Justify question.

Keep it up! Your understanding is improving, I can tell.
 kmpaez
  • Posts: 12
  • Joined: Sep 18, 2017
|
#43295
Why is answer choice A incorrect? I left only answer choices A and B as contenders and had a hard time choosing between them.
 Adam Tyson
PowerScore Staff
  • PowerScore Staff
  • Posts: 5153
  • Joined: Apr 14, 2011
|
#43646
Why does anyone climb Mt. Everest, kmpaez? "Because it's there" is the classic answer, and that can help us here. Answer A is a problem because we don't know that anyone climbs Everest "primarily for spiritual reasons", and in any event our author thinks nobody should do it regardless of their reason. You want money from sponsors? Don't do it! Fame? Don't do it! You want to impress your friends and coworkers, maybe win over the one you love with feats of your derring-do? Stop, go back!

We want to strengthen that NOBODY should climb Mt. Everest. Our evidence is that it is very dangerous and that you won't get any spiritual discovery. Strengthen that connection, fill in that blank, close that gap: if it's very dangerous and you won't get a spiritual discovery, don't do it. That's answer B.
 hassan66
  • Posts: 51
  • Joined: Jul 19, 2018
|
#49768
Is it okay that the stimulus says "spiritual discovery" whereas answer choice B says "spiritual benefits?" I initially chose D but I think I understand why B is correct (or rather, why the other four choices are wrong).

A) be wary of "primary" since spirituality doesn't have to be the main reason and climbing Everest is the main point of the stimulus, not the spirituality so dangerous activities should be the primary focus in the answer choice
C) legal prohibition is outside the scope
D) spiritual experiences are not the focus (like A)
E) the author says that these should be avoided. And this just seems irrelevant.

I know that this is a justify question but it almost seems like a principle question? The umbrella statement is that dangerous activities that are unlikely to result in spiritual benefits should be avoided. The stimulus indicates that risk of death is high so climbing everest could/would fall under the category of dangerous activities. Spiritual discovery could be positive or negative but by saying that if it's unlikely you'll have a positive discovery (or a benefit) then you should avoid this activity. And because the only discoveries that have occurred were of exhaustion and fear, these are not benefits. This adds to the main claim that you should avoid climbing mount everest which falls under the umbrella of not doing dangerous activities that do not result in spiritual benefits.

It's a bit circuitous reasoning; is it valid reasoning?
 hassan66
  • Posts: 51
  • Joined: Jul 19, 2018
|
#57004
Adam Tyson wrote:Good question, asuper, and you have the same two contenders that I had. At first, I thought A was the winner - until I got to D, saw that it was irrelevant, and had to go back and think a little more about A.

D is the correct answer because it doesn't weaken the argument. The efficient ones don't need to improve - that's great, but what does it have to do with helping the ones who are not as productive and may not be as efficient? Answer D focuses on the wrong group of managers, and so it has no bearing on this proposal.

Answer A seems to be attacking the validity of the report and the value of its findings. If the two things being measured were measured using the same criteria, then of course the two will be correlated. You'll get the same answers for both - they are good at both, or they are bad at both, or they are mediocre at both. The correlation will no longer have any value in determining whether one thing causes or influences the other. That's my view of it on my second pass - it's a form of data attack, one of the five classic ways to attack a causal argument.

I hope that helps!
Hi Adam, thank you for fielding all my questions (this and the countless others in recent weeks!), it is extremely helpful in my prep so I sincerely appreciate it. For this question, however, I think you are referring to a different question.

In any case, after reviewing the question after some time, I better understand how to approach this question:

Using the justify formula as was mentioned above you need something that bridges the gap/adds support to an activity having a high risk of danger to then saying you shouldn't climb Everest, something that says something about dangerous activities (which would encompass Everest). When you look at the ACs, only B and C make dangerous activities the focus/subject of the answer, as it should be, but C's addition of legally prohibited is outside the scope so that leaves you with B.
 James Finch
PowerScore Staff
  • PowerScore Staff
  • Posts: 943
  • Joined: Sep 06, 2017
|
#60827
Hi Hassan,

Correct! All Justify questions require an extra premise that will prove the conclusion true, and to do so it has to connect the existing premises to the conclusion, and do so in such a way that the conclusion will 100% be true.

Good job!
 lsnewbie
  • Posts: 12
  • Joined: Aug 31, 2018
|
#60955
Hello,
When I approached this question, I thought it was a strengthen question due to the wording, "most helps to justify". I was under the impression that form of wording was indicative of a strengthen question. I guess it doesn't necessarily hurt to identify a justify question as a strengthen, since justify questions are a form of strengthen questions, but I was just wondering if you had any guidance on how to differentiate between the two moving forward?
Thank you,
JT
 Jay Donnell
PowerScore Staff
  • PowerScore Staff
  • Posts: 144
  • Joined: Jan 09, 2019
|
#62280
Hi JT!

You are correct all across the board here.

First, this question indeed should be identified and approached as a Strengthen (Principle) question, as the modifier of "most helps to justify" implies that the answer may not bring 100% proof to validate the conclusion in the stimulus. A true Justify stem will not lessen the function of the answer with the use of a word like MOST or BEST.

Second, Strengthen questions that involve the use of general principles typically do provide an answer that would satisfy the higher burden of proof needed to qualify as correct for a Justify question. However, it's important to clarify exactly what our mission is so that we can confidently pick an answer that complies with what the question stem demands.

The criteria for a Justify answer is that it 100% proves the conclusion, and the criteria for a Strengthen answer is just that it significantly betters the conclusion, and does so more than any other answer.

If you treat all Strengthen questions as Justify questions, you may run into trouble when no available answer choice brings 100% proof. If you treat all Justify questions as Strengthen questions, you can certainly run into trouble in settling for an answer that merely helps the argument, but doesn't bring in enough support to sufficiently prove the conclusion.

Great work with the eagle-eye, keep up the good work! :-D
 Sambenz
  • Posts: 15
  • Joined: Jun 03, 2020
|
#75913
I would also mention that the order of the premises is highly jumbled making this question quite hard (For me at least).

Their true order would follow like this:

Premise 1: Many climbers regard climbing Everest as the ult. Ach.
Premise 2: The romantic notion of gaining spiritual discovery is dispelled by reports....
Premise 3: The risk of death or injury is very high.
Premise 4: (Answer choice B) - Dangerous activities that are unlikely to result in significant spiritual benefits should be avoided.
Conclusion: therefore climbers should not attempt this climb.

I found this quite tricky because of the order here as well as correctly identifying the conclusion.

Get the most out of your LSAT Prep Plus subscription.

Analyze and track your performance with our Testing and Analytics Package.